State Bank of India PO Preliminary Exam-2015 Question Paper With Answer Key

State Bank of India PO Preliminary Exam-2015
State Bank of India PO Preliminary Exam-2015 Question Paper With Answer Key

State Bank of India PO Preliminary Exam-2015 Held on June 21, 2015

English Language

   Directions-(Q. 1-5) Rearrange the following six sentences (1), (2), (3), (4), (5) and (6) in a proper sequence to form a meaningful paragraph, then answer the given question.

(1) Take for example, the market for learning dancing.

(2) This could never happen if there was a central board of dancing education which enforced strict standards of what will be taught and how much things are to be taught.

(3) The Indian education system is built on the presumption that if something is good for one child, it is good for all children.

(4) More importantly, different teachers and institutes have developed different ways of teaching dancing.

(5) There are very different dance forms that attract students with different tastes.

(6) If however, we can effectively decentralize education, and if the government did not obsessively control what would be the ‘syllabus’ and what will be the method of instruction, there could be an explosion of new and innovative courses geared towards serving various riches of learners.

1. Which of the following should be the LAST(SIXTH) sentence after the rearrangement?

(A)  1

(B)  6

(C)  4

(D)  3

(E)   2

Answer: (E)

2. Which of the following should be the THIRD sentence after the rearrangement?

(A)  1

(B)  2

(C)  6

(D)  4

(E)   5

Answer: (A)

3. Which of the following should be the SECOND sentence after the rearrangement?

(A)  1

(B)  2

(C)  3

(D)  4

(E)   6

Answer: (E)

4. Which of the following should be the FIRST sentence after the rearrangement?

(A)  1

(B)  2

(C)  3

(D)  4

(E)   5

Answer: (C)

5. Which of the following should be the FOURTH sentence after the rearrangement?

(A)  1

(B)  2

(C)  3

(D)  5

(E)   4

Answer: (D)

Directions-(Q. 6-15) Read the following passage carefully and answer the question given. Certain words/phrases, have been given in bold to help you locate them while answering some of the questions.

   Manufacturers of consumer packaged goods (CPG) face two key challenges this year. The first is continued slow of negative growth in people’s disposable incomes. The second is changing consumer attitudes toward products and brands, as the great fragmentation of consumer markets take another turn. In response, companies must dramatically shift the route they take to reach consumers in terms of both product distribution and communications. In many markets, consumer wages have been static for five years now. Even where economies are starting to perform better, the squeeze on after-tax wages, especially for the middle class younger people and families, is depressing consumer spending. Although growth in developing consumer spending. Although growth in developing countries is still better than in the United States and Europe, a slowdown in emerging countries such as China-where many countries has hoped for higher sales has translated quickly into lower-than-expected consumer spending growth.

Meanwhile, what we call the great fragmentation is manifested in consumer behavior and market response. In both developed and emerging markets, there is a wider variety among consumers now than at any time in the recent past. Growth is evident both at the top of the market (where more consumers are spending for higher-quality food and other packaged goods) and at the lower end (where an increasing number of consumers are concentrating on value). But the traditional middle of the market is shrinking.

   Further, individual consumer behavior is more pluralistic. We are used to seeing, for example, spirits buyers purchasing a premium band in a bar, a less costly label at home for personal consumption ad yet another when entertaining guests. But his type of variegated shopping has now spread to the grocery basket as well. Fewer consumers are making one big stocking-up trip each week. Instead, shoppers are visiting a premium store and a discounter as well as a super-market, in multiple weekly stops-in addition to making frequent purchases online. During recession, more shoppers became inclined to spend tie hunting for bargains and as some traditional retailers either went out of business or shuttered down, retail space was freed up and was often filled by convenience stores, specialty shops, and discounters.

A decade ago, VCPG companies had only a handful of sales channels to consider supermarkets, convenience stores, hypermarkets in advanced economies and traditional small and large retailers in emerging countries. Since, then various discounters have made significant inroads, including no frills. low variety outlets, such as Europe’s Aldi and Lidi, which sell a limited range of private—label grocery items in smaller stores and massive warehouse clubs, such as Costco and Sam’s club, which initially operated solely in the U.S. but are now expanding internationally. In addition, dollar stores, specialized retailers, and online merchants are having an impact on the CPG landscape. Economising consumers have been pleasantly, surprised by the savings generated by spreading their business among multiple channels, as well as by the variety and product quality they find. The result has been greater demand for more products and brands, with different sizes, packaging and sales methods, At most CPG companies, SKUs are proliferating despite there being little increase in overall consumption. A better outcome can be seen at smaller food and beverage suppliers, which are benefiting from consumer demand for variety and authenticity. A recent report found that in the U.S., small manufacturers (with revenues of less than US $1 billion) grew at twice the compound annual rate of large manufacturers (with revenues of more than $3 billion) between 2009 and 2012.

Consumers’ media usage has also fragmented with the rise of digital content and the proliferation of online devices. Each channel-from the Web, Mobile and social sites for radio, TV and print has its own requirements audience appear and economics, needing specialized attention. But at the same time, media campaigns need to be closely co-ordinated for effective consumer messaging.

Collectively, these shifts challenge the way CPG companies manage their brand and business portfolios and call for a rethinking of their go-to-market approach, with an emphasis on analytics. Our work with INSEAD shows that among business leaders, applying analytics-especially for tracking consumer behavior and product and promotional performance-considered one of the most effective ways to improve results and outpace the competition. But it’s not just about insight. It’s also about using the insight wisely to determine how to manage costs. The more knowledgeable about customer needs and preferences a company is, the smarter and more focused it must be in managing its own economics to cost-effectively deliver both variety and value to be squeezed consumer.

6. The central theme of the given passage is-

(A)  The shrinking market

(B)  Shift towards offering luxury goods to consumers

(C)  Products to offer consumers with squeezed pockets

(D)  To highlight products consumed by the middle class

(E)   Gaining insight into changing consumer behavior towards CPGs

Answer: (E)

7. In the context of the passage, which of the following brands existed otherwise but is now being manifested in buying groceries as well?

(A)  Consumers purchasing the same products for over a period of time

(B)  Consumer willing to purchase goods for a longer period of time

(C)  Consumers preferring luxury goods over regular goods

(D)  Consumers are more aware of their rights

(E)   Consumers prefer buying goods from a variety of stores

Answer: (E)

8. Which of the following is most nearly the OPPOSITE in meaning to the word ‘DEPRESSING’ as used in the passage?

(A)  Encouraging

(B)  Sunny

(C)  Doubtful

(D)  Light

(E)   Nil

Answer: (A)

9. As mentioned in the passage, CPG companies may have to reassess their present strategies of operating to-

(A)  retain their customers.

(B)  keep pace with changing consumer preferences as they have access to multiple media channels.

(C)  make more cost-effective decisions.

(A)  Only (a)

(B)  Only (b)

(C)  All the three (a), (b) and (c)

(D)  Only (c)

(E)   Only (a) and (b)

Answer: (C)

10. Which of the following is true in the context of the passage?

(A)  In the U.S., during the three year period after 2009, small manufacturers did not fare well as compared to their larger counterparts

(B)  Impact on disposable incomes of people barely affects the CPG manufacturing industry

(C)  Post-tax wages, especially for the middle class, are one of the critical factors which have reduced spending behavior of consumers

(D)  CPG have always been a favourite among consumers

(E)   None of the given options is true

Answer: (C)

11. Which of the following correctly explains the meaning of phrase, ‘a handful of’ as used in the passage?

(A)  Boundless

(B)  Planned

(C)  Satisfactory

(D)  Limited

(E)   Imperfect

Answer: (D)

12. As mentioned in the passage, one of the most critical factors that aids in catering to the needs of consumers is-

(A)  persuading them to purchase goods produced by the organization

(B)  assess their requirements and appropriately plan to meet them

(C)  offering them products that an organization regularly manufactures

(D)  concentrating only on being aware about changing preferences of consumers

(E)   None of the given options

Answer: (D)

13. Which of the following is most nearly the SAME in meaning to the word ‘SHRINKING’ as used in the passage?

(A)  Developing

(B)  Annoying

(C)  Narrowing

(D)  Wasting

(E)   Rising

Answer: (C)

14. Which of the following is most nearly the SAME in meaning to the word ‘VARIEGATED’ as used in the passage?

(A)  Diverse

(B)  Composite

(C)  Strong

(D)  Narrow

(E)   Valued

Answer: (A)

15. Which of the following is most nearly the OPPOSITE in meaning to the word ‘SHUTTERED’ as used in the passage?

(A)  Closed

(B)  Retail

(C)  Flourished

(D)  Gratified

(E)   Nearest

Answer: (C)

Directions-(Q. 16-20) Read this sentence to find out whether there is any grammatical mistake / error in it. The error, if any, will be in one part of the sentence. Mark the part with the error as your answer. If there is no error, mark ‘No error’ as your answer. (Ignore, errors of punctuation, if any)

16. After his term in the Department of Defence / he was appointed Secretary of Energy / a job in which he was strongly supported the use / of alternate sources of energy such as nuclear energy.

(A)  After his term in the Department of Defence

(B)  He was appointed Secretary of Energy

(C)  A job in which he was strongly supported the use

(D)  Of alternate sources of energy such as nuclear energy

(E)   No error

Answer: (C)

17. As GDP growth is half of what it/ is just a few years ago, the country/ desperately needs to cut red tape and improve/ infrastructure to boost investment and growth.

(A)  As GDP growth is half of what it

(B)  Is just a few years ago, the country

(C)  Desperately needs to cut red tape and improve

(D)  Infrastructure to boost investment and growth

(E)   No error

Answer: (B)

18. When elections in the country due shortly/ people are determined to register as voters and to vote for / candidates based on their track record and the programmes/ that they intend to implement for the betterment of the country.

(A)  When elections in the country due shortly

(B)  People are determined to register as voters and to vote for

(C)  Candidates based on their track record and the programmes

(D)  That they intend to implement for the betterment of the country

(E)   No error

Answer: (A)

19. At least a quarter of the World’s corals have lost/over the past twenty-five years and as climate change warms/ the oceans the additional carbon dioxide will make / the water more acidic further destroying coral reefs.

(A)  Atleast a quarter of the World’s corals have lost

(B)  Over the past twenty-five years and as climate change warms

(C)  The oceans the additional carbon dioxide will make

(D)  The water more acidic further destroying coral reefs

(E)   No error

Answer: (A)

20. The Prime Minister’s speech laid out how/ it is important to encourage female participated / in the economy yet the percentage of female lawmakers in / the lower house of parliament has fallen to 8 per cent.

(A)  The Prime Minister’s speech laid out now

(B)  It is important to encourage female participated

(C)  In the economy yet the percentage of female lawmakers in

(D)  The lower house of parliament has fallen to 8 per cent

(E)   No error

Answer: (B)

Directions-(Q. 21-25) The question has two blanks each blank indicating that something has been omitted. Choose the set of words for each blank that best fits the meaning of the sentence as a whole.

21. Three unknown assailants………….. on motor-cycles and opened ………… barely a few metres from where the actor was shooting.

(A)  came, shooting

(B)  rode, rounds

(C)  arrived, fire

(D)  approached, hitting

(E)   climbed, shots

Answer: (C)

22. The police ………….. in……….. a car theft case within a matter of two hours of registration of the complaint.

(A)  achieved, solving

(B)  victorious, catching

(C)  famed, arresting

(D)  succeeded, cracking

(E)   failed, assigning

Answer: (D)

23. The health ministry has approved major expansion of post-graduate seats in key departments of AIIMS to ……… the severe ………. of specialists across the country.

(A)  address, dearth

(B)  correct, loss

(C)  improve, damage

(D)  rectify, limits

(E)   good, shortage

Answer: (A)

24. With the ……….. wedding season, people are leaving no stone unturned to make their wedding cards look ………..

(A)  prolonged, beyond

(B)  ongoing, special

(C)  instant, great

(D)  sudden, while

(E)   estimated, legible

Answer: (B)

25. The tourism data ……….. that Indians are ………… the biggest spenders while on holidays.

(A)  says, within

(B)  reveal, amongst

(C)  proclaims, between

(D)  states, surrounded

(E)   speaks, amid

Answer: (B)

Directions-(Q. 26-30) In the given passage, there are blanks each of which has been numbered. Against each five words are suggested, one of which fits the blank appropriately. Find out the appropriate word in each case.

   A strong economy would improve livelihoods and give hope for the future, but the lack of reliable electricity in the country has contributed to holding back prosperity. Over the past five years, GDP growth has averaged 3 per cent, too …(26).. to fight poverty and create jobs. Demand for power outstrips what the country can produce and …(27)… Planned and unplanned electricity outages of 12 to 16 hours daily nationwide …(28)… business, aggravate unemployment and spark angry protests. The country has …(29)… energy resources-an estimated 186 billion tones of coal, over 100,000 megawatts of hydro potential and wind potential of upto 3,46,000 megawatts. But the technology and investment to …(30)… these resources are limited. An overreliance on imported fuel has exposed the country to high oil prices and there isn’t enough money to keep the system-a mix of private and state run enterprises-running or to fix faulty infrastructure that leaks electricity. A resolution of power crisis is thus very critical to survival of the country.

26.

(A)  much

(B)  low

(C)  required

(D)  tough

(E)   minimal

Answer: (B)

27.

(A)  represent

(B)  sufficient

(C)  adequate

(D)  amount

(E)   deliver

Answer: (E)

28.

(A)  run

(B)  happens

(C)  plans

(D)  effect

(E)   hurt

Answer: (D)

29.

(A)  ample

(B)  fix

(C)  worst

(D)  frequent

(E)   paucity

Answer: (A)

30.

(A)  lack

(B)  solve

(C)  waste

(D)  exploit

(E)   advantage

Answer: (D)

Quantitative Aptitude

31. As present, Pia is 5 years older to Ray. The respective ratio between the present ages of Pia and Mini is 3:4. At present, Ray is 14 years younger to Mini. What is Ray’s present age?

(A)  16 years

(B)  20 years

(C)  14 years

(D)  18 years

(E)   24 years

Answer: (D)

Directions-(Q. 32-36) In the question two equations numbered (I) and (II) are given. You have to solve both the equations and mark the appropriate answer.

32. (I) 4x2 + 17x + 15 = 0

(II) 3y2 + 19y + 28 = 0

(A)  x ≥ y

(B)  x ≤ y

(C)  x > y

(D)  x < y

(E)   relationship between x and y cannot be established

Answer: (E)

33. (I) 3x2 – 17x + 22 = 0

(II) 5y2 – 21y + 22 = 0

(A)  x ≥ y

(B)  x ≤ y

(C)  x > y

(D)  x < y

(E)   relationship between x and y cannot be established

Answer: (A)

34. (I) 3x2 + 11x + 10 = 0

(II) 2y2 + 13y + 21 = 0

(A)  x ≥ y

(B)  x ≤ y

(C)  x > y

(D)  x < y

(E)   relationship between x and y cannot be established

Answer: (C)

35. (I) 3x2 + 13x + 14 = 0

(II) 8y2 + 26y + 21 = 0

(A)  x ≥ y

(B)  x ≤ y

(C)  x > y

(D)  x < y

(E)   relationship between x and y cannot be established

Answer: (D)

36. (I) 3x2 – 14x + 15 = 0

(II) 15y2 – 34y + 15 = 0

(A)  x ≥ y

(B)  x ≤ y

(C)  x > y

(D)  x < y

(E)   relationship between x and y cannot be established

Answer: (A)

Directions-(Q. 37-41) Study the table to answer the given question.

   Data related to expenses by companies A, B, C, D and E in the April 2012.

(Note- Other than the mentioned expenses under the given heads, there may also be some other expense. For calculation purposes, please consider the given expense heads only.)

37. What is the respective ratio between the marketing expense of company A and the marketing expenses of company C?

(A)  34 : 35

(B)  32 : 33

(C)  35 : 36

(D)  36 : 37

(E)   37 : 38

Answer: (D)

38. Total expenses of Company E in October 2012 was 20% more than the total given expenses of the same company in April 2012. How much was the Marketing expenses of the same Company in October 2012, if it was 25% of the total expenses of the same month? (in Rs. Lakhs)

(A)  27

(B)  24

(C)  26

(D)  21

(E)   18

Answer: (D)

39. Marketing expenses of company D is less than marketing expenses of Company B, by approximately what per cent?

(A)  20

(B)  40

(C)  39

(D)  59

(E)   29

Answer: (E)

40. What is the average total expense of all the companies ? (in Rs. Lakhs)

(A)  75.25

(B)  75.8

(C)  75

(D)  72.8

(E)   72.25

Answer: (B)

41. What is the difference between the total expenses made by company C in legal and marketing together and the total expenses made by company E in legal and marketing together? (in Rs. Lakh)

(A)  24.5

(B)  23.5

(C)  22.9

(D)  23

(E)   23.9

Answer: (E)

Directions-(Q. 42-51) What approximate value will come in place of the question mark (?) in the given question ? (You are not expected to calculate the exact value).

42. 

(A)  14

(B)  22

(C)  2

(D)  16

(E)   8

Answer: (B)

43. 

(A)  670

(B)  570

(C)  710

(D)  510

(E)   610

Answer: (A)

44. 

(A)  625

(B)  400

(C)  25

(D)  900

(E)   225

Answer: (E)

45. 14.85% of 679 + 19.9% of 219.89=?

(A)  115

(B)  145

(C)  65

(D)  105

(E)   85

Answer: (B)

46. 1441 ÷ ? + 149.98 ×99 = 3006 – 254.91 –

(A)  35

(B)  15

(C)  25

(D)  45

(E)   3

Answer: (E)

47. 11 31 69 131 223 ?

(A)  257

(B)  351

(C)  349

(D)  231

(E)   254

Answer: (B)

48. 12 15 20 27 36 ?

(A)  47

(B)  59

(C)  43

(D)  49

(E)   51

Answer: (A)

49. 5 9 18 34 59 95 ?

(A)  272

(B)  168

(C)  116

(D)  148

(E)   144

Answer: (E)

50. 19 20 16 25 9 ?

(A)  32

(B)  38

(C)  34

(D)  42

(E)   48

Answer: (C)

51. 65 70 77 86 97 ?

(A)  110

(B)  121

(C)  100

(D)  112

(E)   90

Answer: (A)

52. A bag contains 5 red balls, 6 yellow balls and 3 green balls. If two balls are picked at random, what is the probability that either both are red or both are green in colour?

(A)  3/7

(B)  5/14

(C)  1/7

(D)  2/7

(E)   3/14

Answer: (C)

53. In a 140 litres of mixture of milk and water, percentage of water is only 30%. The milkman gave 20 litres of this mixture to a customer. Then he added equal quantities of pure milk and water to the remaining mixture. As a result the respective ratio of milk and water in the mixture became 2:1. What was the quantity of milk added? (in litres)

(A)  12

(B)  16

(C)  18

(D)  8

(E)   10

Answer: (C)

54. A boat can travel 4.2 km upstream in 14 minutes. If the respective ratio of speed of the boat in still water and sped of the stream is 7:1. How much time will the boat take to cover 17.6 km downstream? (in minutes)

(A)  52

(B)  44

(C)  48

(D)  36

(E)   54

Answer: (B)

55. A starts a business with a capital of Rs 1500. B joins the business 6 months later from the start of the business and C joins the business 8 months later from the start of the business. End of the year their respective share in profit was in ratio of 5 : 3 : 3. What is the sum of amount put in the business by B and C together?

(A)  Rs 3300

(B)  Rs 3500

(C)  Rs 4200

(D)  Rs 4800

(E)   Rs 4500

Answer: (E)

56. The length of a rectangle is 4m more than the side of a square and the breadth of the rectangle is 4m less than side of the same square. If the area of the square is 576 sq.m., what is the area of the rectangle ? (in sq.m)

(A)  549

(B)  545

(C)  557

(D)  560

(E)   551

Answer: (D)

57. A sells a bicycle to B at a profit of 20% and B sells it to C at a profit of 25%. If C pays Rs 1500, what did A pay for it?

(A)  Rs 1000

(B)  Rs 1500

(C)  Rs 600

(D)  Rs 2000

(E)   Rs 500

Answer: (A)

58. 10 men can complete a work in 7 days. But 10 women need 14 days to complete the same work. How many days will 5 men and 10 women need to complete the work?

(A)  5

(B)  6

(C)  7

(D)  8

(E)   9

Answer: (C)

59.  A sum of Rs 6400 earns a compound interest of Rs 1008.80 in 18 months, when the interest is reckoned half yearly. Find the rate of interest-

(A)  20%

(B)  15%

(C)  10%

(D)  25%

(E)   30%

Answer: (C)

Directions-(Q. 60-64) Refer to the graph and answer the given question.

60. Number of hats sold by store M during January is what per cent of total number of hats sold by the store N during March and April together?

(A) 

(B) 

(C) 

(D) 

(E) 

Answer: (D)

61. What is the average number of hats sold by store N during January, March, May and June?

(A)  535

(B)  525

(C)  515

(D)  500

(E)   530

Answer: (B)

62. Stores M and N sells only two types of hats : Fedora Hats and Trilby Hats. If the respective ratio of total number of Fedora hats to total number of Trilby hats sold by stores M and N together during March is 9 : 5, what is the total number of Fedora hats sold by stores M and N together during March?

(A)  990

(B)  900

(C)  720

(D)  630

(E)   810

Answer: (E)

63. If the total number of hats sold by stores M and N together in July is 15% more than the total number of hats sold by the same stores together during June, what is the total number of hats sold by the same stores together during July?

(A)  1298

(B)  1316

(C)  1356

(D)  1284

(E)   1331

Answer: (E)

64. Number of hats sold by store M increased by what per cent from February to May?

(A)  75

(B)  55

(C)  65

(D)  70

(E)   60

Answer: (C)

65. Monthly salary of Dex is 1/4th his father’s monthly salary. Dex’s sister’s monthly salary is 2/5th of their father’s monthly salary. Dex’s sister pays Rs 12800, which is 1/4th of her monthly salary as study loan. Savings and expenses made out of the monthly salary, by Dex is in the respective ratio 3 : 5. How much does Dex save each month?

(A)  Rs 12000

(B)  Rs 10600

(C)  Rs 10400

(D)  Rs 12600

(E)   Rs 12400

Answer: (A)

Reasoning Ability

   Directions-(Q. 66-70) Study the information carefully and answer the given question.

   A, B, C, D, E, F, G and H are sitting around a circular area of equal distance between each other, but not necessarily in the same order. Some of the people are facing the centre while some face outside. (i.e., in a direction opposite to the centre).

Note-Same direction means if one person is facing the centre then the other also faces the centre and vice-versa Opposite direction means that if one person is facing the centre then the other faces outside and vice-versa.

D sits third to the right of B. E sits seconds to the left of B. Immediate neighbours of B face the same direction (i.e., if one neighbor faces the centre the other neighbor also faces the centre and vice-versa) C sits second to the left of E. E faces the centre. F sits third to the right of C. G sits second to the left of H. H is not an immediate neighbor of B. G faces the same direction as D. Immediate neighbours of E face opposite directions (i.e., if one neighbor faces the centre the other neighbor faces outside and vice-versa). Immediate neighbours of F face opposite direction (i.e., if one neighbor faces the centre the other neighbor face outside and vice-versa).

66. How many people in the given arrangement face the centre?

(A)  Two

(B)  Three

(C)  One

(D)  Four

(E)   Five

Answer: (E)

67. Which of the following is true regarding A as per the given seating arrangement?

(A)  H sits second to the left of A

(B)  A faces outside

(C)  Only two people sit between A and B

(D)  C is the one of the immediate neighbours of A

(E)   Only three people sit between A and G

Answer: (B)

68. Four of the following fiver are alike in a certain way based on the given seating arrangement and so form a group. Which is the one that does not belong to that group?

(A)  F

(B)  H

(C)  B

(D)  G

(E)   D

Answer: (B)

69. What is E’s position with respect to H?

(A)  Third to the left

(B)  To the immediate left

(C)  To the immediate right

(D)  Second to the right

(E)   Third to the right

Answer: (B)

70. Who sits second to the left of G?

(A)  H

(B)  A

(C)  B

(D)  Other than those given as options

(E)   F

Answer: (A)

Directions-(Q. 71-75) Study the given information carefully to answer the given question.

   A, B, C, D, K, L and M live on seven different floors of a building but not necessarily in the same order. The lowermost floor of the building is numbered one, the one above that is numbered two and so on till the topmost floor is numbered seven. Each one of them also likes different games namely. Snooker, Badminton, Chess, Ludo, Cricket, Hockey and Polo (but not necessarily in the same order.)

Only three people live between B and K. B lives on one of the floors above K. K does not live on the lowermost floor. Only one person lives between B and the one who like Chess. The one who likes Polo lives on the of the even numbered floors above the one who like Chess.

Only two people live between M and the one who likes Chess. The one who likes Snooker lives immediately above M. A lives immediately above L. A does not like Chess.

The one who likes Ludo lives on one of the odd numbered floors below L. M does not live Ludo. D lives on one of the Floors above C. Only one person lives between the one who likes Cricket and the one who likes Hockey. D does not like Cricket. M does not like Badminton.

71. Which of the following games does B like?

(A)  Snooker

(B)  Ludo

(C)  Polo

(D)  Badminton

(E)   Chess

Answer: (D)

72. Who amongst the following lives on the floor numbered 4?

(A)  The one who likes Hockey

(B)  The one who likes Chess

(C)  A

(D)  L

(E)   B

Answer: (A)

73. Which of the following statements is true with respect to the given arrangement?

(A)  Only two people live between K and M

(B)  The one who likes Hockey lives immediately above K

(C)  C likes Chess

(D)  C lives on an even numbered floor

(E)   None of the given options is true

Answer: (B)

74. If all the people are made to sit in alphabetical order from top to bottom, the positions of how many people will remain unchanged?

(A)  None

(B)  Three

(C)  Two

(D)  One

(E)   Four

Answer: (D)

75. Which of the following combinations is true with respect to the given arrangement?

(A)  Polo-C

(B)  Ludo-B

(C)  Cricket-K

(D)  Chess-L

(E)   Snooker-A

Answer: (D)

76. A person starts walking from his office towards a party has. He walks for 30 m towards East. He takes a 90 right turn and walks for 15 m. He again takes a 90° right turn and walks for 15 m. He again takes a 90° right turn, and walks for another 20 m. He then walks for 25 m after taking a 90 left turn. Turning 90° towards his right he walks for 10 m to reach the party hall. How far and in which direction is the party hall from his office?

(A)  40 m towards West

(B)  40 m towards South

(C)  45 m towards South

(D)  45 m towards North

(E)   40 m towards North

Answer: (B)

Directions-(Q. 77-81) In this question, two/three statements followed by two conclusions numbered I and II have been given. You have to take the given statements to be true even if they seem to be at variance from the commonly facts and then decide which of the given conclusions logically follows from the given statements disregarding commonly known facts.

77. Statements:

Some wins are trophies.

Some trophies are cups.

No cup is a prize.

Conclusions:

(I) Atleast some cups are wins.

(II) All prizes being trophies is a possibility.

(A)  Only conclusion I is true

(B)  Only conclusion II is true

(C)  Either conclusion I or II true

(D)  Both conclusion I and II are true

(E)   Neither conclusion I nor II is true

Answer: (B)

78. Statements:

No layer is a coat.

All coats are deposits.

All deposits are sheets.

Conclusions:

(I) All coats are sheets.

(II) All deposits can never be layers.

(A)  Only conclusion I is true

(B)  Only conclusion II is true

(C)  Either conclusion I or II true

(D)  Both conclusion I and II are true

(E)   Neither conclusion I nor II is true

Answer: (D)

79. Statements:

Some flats are apartments.

No apartment is a hall.

Some halls are rooms.

Conclusions:

(I) Atleast some rooms are flats.

(II) No apartment is a room.

(A)  Only conclusion I is true

(B)  Only conclusion II is true

(C)  Either conclusion I or II true

(D)  Both conclusion I and II are true

(E)   Neither conclusion I nor II is true

Answer: (E)

80. Statements:

Some wins are trophies.

Some trophies are cups.

No cup is a prize.

Conclusions:

(I) No trophy is a prize.

(II) No prize is a win.

(A)  Only conclusion I is true

(B)  Only conclusion II is true

(C)  Either conclusion I or II true

(D)  Both conclusion I and II are true

(E)   Neither conclusion I nor II is true

Answer: (E)

81. Statements:

Some codes are secrets.

All secrets are puzzles.

Conclusions:

(I) All secrets being codes is a possibility

(II) Atleast some puzzles are codes.

(A)  Only conclusion I is true

(B)  Only conclusion II is true

(C)  Either conclusion I or II true

(D)  Both conclusion I and II are true

(E)   Neither conclusion I nor II is true

Answer: (B)

82. Point A is 40 m to the north of point B. Point C is 20 m to the west of point A. Point D is 30 m to the south of point C. Point E is exactly midway between points D and F in such a manner that Point D, E and F form a horizontal straight line of 40 m. Point F is to the east of point D. Point G is 30 m to the north of point F. How far and in which direction is point G from point A?

(A)  30 m towards west

(B)  40 m towards north

(C)  20 m towards west

(D)  20 m towards east

(E)   30 m towards east

Answer: (D)

83. Statements : L = P ≤ W < V ≤ K ≥ Q; B < L; K = M

Conclusions : (I) B < V

(II) M > P

(A)  Only conclusion I is true

(B)  Only conclusion II is true

(C)  Either conclusion I or II is true

(D)  Both conclusion I and II are true

(E)   Neither conclusion I nor II is true

Answer: (D)

84. Statements : L = P ≤ W < V ≤ K ≥ Q; B < L; K = M

Conclusion: (I) L ≥ Q

(II) W = M

(A)  Only conclusion I is true

(B)  Only conclusion II is true

(C)  Either conclusion I or II is true

(D)  Both conclusion I and II are true

(E)   Neither conclusion I nor II is true

Answer: (E)

85. Statements: R ≤ U = B < S; B ≤ X

Conclusions : (I) X > R (II) X = R

(A)  Only conclusion I is true

(B)  Only conclusion II is true

(C)  Either conclusion I or II is true

(D)  Both conclusion I and II are true

(E)   Neither conclusion I nor II is true

Answer: (C)

86. Statements: C > U ≤ S < T = O > D ≥ Y; Z = O ≤ P

Conclusions: (I) U > D (II) S < P

(A)  Only conclusion I is true

(B)  Only conclusion II is true

(C)  Either conclusion I or II is true

(D)  Both conclusion I and II are true

(E)   Neither conclusion I nor II is true

Answer: (B)

87. Statements: C > U ≤ S < T = O > D ≥ Y; Z = O ≤ P

Conclusions: (I) Z > Y (II) C < O

(A)  Only conclusion I is true

(B)  Only conclusion II is true

(C)  Either conclusion I or II is true

(D)  Both conclusion I and II are true

(E)   Neither conclusion I nor II is true

Answer: (A)

Directions-(Q. 88-90) Study the following information and answer the given question.

   J is the sister of T. T is married to D. D is the father of R.

M is the son of H. T is the mother-in-law of H.

D has only one son and no daughter.

J is married to B. L is the daughter of B.

88. How is L related to T?

(A)  Niece

(B)  Sister

(C)  Cannot be determined

(D)  Daughter

(E)   Mother

Answer: (A)

89. How is M related to D?

(A)  Nephew

(B)  Uncle

(C)  Brother

(D)  Son

(E)   Grandson

Answer: (E)

90. How is J related to R?

(A)  Sister

(B)  Aunt

(C)  Mother-in-law

(D)  Mother

(E)   Cannot be determined

Answer: (B)

Directions-(Q. 91-95) Study the given information carefully to answer the given question in a certain code language.

   ‘efforts required in teaching’ is written as ‘dp kj te hy’

‘nothing kids is tough’ is written as ‘dp op cs mb’

kids heart is huge’ is written as ‘cs re mb gr’

‘task required huge efforts’ is written as ‘hy kj re ba’

(All codes are two letter codes only)

91. In the given code language, what does the code ‘kj stand for?

(A)  in

(B)  either ‘required’ or ‘efforts’

(C)  either ‘huge’ or ‘kids’

(D)  task

(E)   huge

Answer: (B)

92. What is the code for ‘teaching’ in the given code language?

(A)  hy

(B)  te

(C)  op

(D)  other than those given as options

(E)   kj

Answer: (D)

93. What is the code for ‘kids’ in the given code language?

(A)  either ‘mb’ or ‘cs’

(B)  either ‘op’ or ‘gr’

(C)  dp

(D)  other than those given as options

(E)   re

Answer: (A)

94. What will be the code for ‘in task’ in the given code language?

(A)  te hy

(B)  dp kj

(C)  ba kj

(D)  ba te

(E)   other than those given as options

Answer: (D)

95. What may be the possible code for ‘quite tough heart’ in the given code language?

(A)  uc ba re

(B)  gr uc re

(C)  op uc gr

(D)  op kj hy

(E)   op gr kj

Answer: (C)

Directions-(Q. 96-100) Study the following information to answer the given question.

   Eight friends, E, F, G, H, L, M, N and O are seated in a straight line, facing North, but not necessarily in the same order.

O sits at the extreme right end of the line. Only four people sit between O and G.

Both F and M are immediate neighbours of G.

Only two people sit between M and L. L is not an immediate neighbor of O.

N sits second to left of E.

96. What is the position of L with respect to G?

(A)  Third to the right

(B)  To the immediate left

(C)  Second to the right

(D)  Fourth to the right

(E)   Second to the left

Answer: (E)

97. Based on the given arrangement, which of the following is true with respect to N?

(A)  Only three persons sit between N and O

(B)  None of the given options is true

(C)  Only one person sits to the right of N

(D)  E sits to immediate right of N

(E)   Both L and F are immediate neighbours of N

Answer: (B)

98. Who amongst the following people represents the person seated at the extreme left end of the line?

(A)  E

(B)  N

(C)  L

(D)  F

(E)   H

Answer: (C)

99. How many persons are seated between O and E?

(A)  Two

(B)  Three

(C)  None

(D)  Four

(E)   One

Answer: (C)

100. Who amongst the following sit exactly between M and L?

(A)  E, N

(B)  F, O

(C)  F, G

(D)  E, G

(E)   H, N

Answer: (C)

State Bank of India PO Preliminary Exam-2016 Question Paper With Answer Key

State Bank of India PO Preliminary Exam-2016
State Bank of India PO Preliminary Exam-2016 Question Paper With Answer Key

State Bank of India PO Preliminary Exam-2016 Held on July 2016

Solved Paper

English Language

Directions-(Q. 1-10) Read the following passage carefully and answer the questions. Certain words/phrases are given in bold to help you locate them while answering some of the questions.

   Until the 1960s boys spent longer and went further in school than girls, and were more likely to graduate from university. Now, across the rich world and in a growing number of poor countries, the balance has titled the other way. Policymakers once fretted about girls’ lack of confidence in science, but his is changing. Sweden has commissioned research into its ‘boy crisis’. Australia has devised a reading programme called ‘Boys, Blokes, Books and Bytes’. In just a couple of generations, one gender gap has closed, only for another to open  up. The reversal is laid out in a report published on March 5th by the OECD a Paris-based rich-country think-tank. Boys’ dominance just about endures in maths : at age 15 they are, on average, the equivalent of thee months’ schooling ahead of girls. In science the results are fairly even. But in reading, where girls have been ahead for some time, ,a gulf has appeared. In all G4 countries and economies in the study, girls outperform boys. The average gap is equivalent to an extra year of schooling. The OECD deems literacy to be the most important skill that it assesses, since further learning depends on it. Sure enough, teenage boys are 50% more likely than girls to fail to achieve basic proficiency in any of maths, reading and science.

Youngesters in this group, with nothing to build on or shine at, are prone to drop out of school altogether. To see why boys and girls fare so differently in the classroom, first look at what they do outside it. The average 15-year old girl devotes five-and-a-half hours a week to homework, an hour more than the average boy, who spend more time playing video games and trawling the internet. Three-quarters of girls read for pleasure, compared with little than half of boys Reading rates are falling everywhere as screens draw eyes from pages, but boys are giving up faster. The OECD found that, among boys who do as much homework as the average girl, the gender gap in reading fell  by nearly a quarter.

Once in the classroom, boys long to be out of it. They are twice as likely as girls to report that school is a ‘waste of time’, and more often turn up late. Just as a teacher used to struggle to persuade girls that science is not only for men, the OECD now urges parents and policymakers to steer bys away from a version of masculinity that ignores academic achievement. Boys’ disdain for school might have been less irrational when there were, plenty of jobs for un-educated men. But those days have long gone. It may be that a bit of swagger helps in maths, where confidence plays a part in boys’ lead (though it sometimes extends to delusion : 12% of boys told the OECD that they are familiar with the mathematical concept of ‘subjunctive sealing’, a red herring that fooled only 7% of girls.) But their lack of self-discipline drives teachers crazy. The OECD found that boys did much better in its anonymised tests than in teachers assessments. What is behind this discrimination ? One possibility is that teachers mark up students who are polite, eager and stay out of flights, all attributes that are more common among girls. In some countries, academic points can even be docked for bad behaviour.

1. Choose the word which is Opposite in meaning to the word Docked given in bold as used in the passage.

(A)  Raised

(B)  Stopped

(C)  Widened

(D)  Flown

(E)   Wharf

Answer: (A)

2. According to the passage, what can be said about the school education today?

(A)  Science education is deteriorating rapidly

(B)  Online education can easily address its problems such as shortage of teaching staff

(C)  It fosters rote learning instead of creative thinking

(D)  The amount of homework for children is prohibitive

(E)   Girls are doing better at school as compared to boys on some parameters

Answer: (E)

3. Choose the word/group of words which is most nearly the same in meaning as the word Draw given in bold as used in the passage.

(A)  Sketch

(B)  Tie

(C)  Raffle

(D)  Represent

(E)   Divert

Answer: (E)

4. Which of the following is TRUE in the context of the passage?

(A)  Boys perform better than girls on subjective teacher assessments

(B)  Efforts to improve representation of girls in education have had success

(C)  B and large teachers are female and they discriminate against boys

(D)  Education in rich countries needs to be subsidized to reduced dropout numbers

(E)   None of the given statements is true in the context of the passage

Answer: (B)

5. Choose the word/group of words which is most nearly the Same in meaning as the word Prone given in bold as used in the passage.

(A)  Unconscious

(B)  Flat

(C)  Likely

(D)  Lifeless

(E)   Opinionated

Answer: (C)

6. Which of the following factors can have an impact on results of boys in school?

(1) Perceptions of teachers.

(2) Societal attitude towards educational achievement and boys.

(3) Overconfidence of male students.

(A)  Only 1

(B)  Only 2

(C)  All 1, 2 and 3

(D)  Only 1 and 3

(E)   Only 1 and 2

Answer: (D)

7. What do the OECD statistics in the passage indicate?

(A)  Schools dropout rates among boys are higher in developing countries than in rich ones

(B)  Despite the perception that girls are doing better than boys in school, the same is not true

(C)  Today boys are more at risk than girls in terms of educational achievement in developed countries

(D)  Enrolment of girls in schools has doubled while that of boys has fallen

(E)   By and large teenagers have very low educational achievement in rich countries

Answer: (D)

8. Choose the word which is Opposite in meaning to the word Delusion given in bold as used in the passage.

(A)  Myth

(B)  Superstition

(C)  Precipitating

(D)  Reality

(E)   Familiarity

Answer: (D)

9. Which of the following best describes the author’s opinion about the ‘boy crisis’?

(A)  It is not as much of a problem as it is made out to be

(B)  Policymakers should address the issue of ‘uneducated’ boys as it will impact boys employment subsequently

(C)  It can be addressed by implementing quotas at  university level

(D)  It is a rich country phenomenon and can be easily addressed through increased funding for schools

(E)   None of the given options

Answer: (B)

10. Which of the following is an appropriate Title for the passage?

(A)  Finding the Glass Ceiling

(B)  Men Storming Up the Irony Tower

(C)  Pay and Job Flexibility

(D)  Attention! A New Gender Gap

(E)   A Broken Safety Net

Answer: (D)

Directions-(Q. 11-15) Read this sentence to find out whether there is any grammatical error in it. The error, if any, will be in one part of the sentence. Mark the part with the error as your answer. If there is no error, mark ‘No error’ as your answer. (Ignore the errors of punctuation, if any).

11. The fare will be calculated (A)/ on the basis of (B)/ expected travel time distance (C)/ and traffic where applied. (D)/ No error (E)

Answer: (D)

12. Junior colleges sees (A)/ marginal violations in (B)/ minimum score cap for arts, (C)/ science and commerce streams. (D)/ No error (E)

Answer: (A)

13. The actor has (A)/ filed a case (B)/ against the director and (C)/ has sought a written apology. (D)/ No error (E)

Answer: (E)

14. The practice of big pharma companies (A)/ offering kickbacks to (B)/ prescribing physicians may not be (C)/ a breach of ethics. (D)/ No error (E)

Answer: (E)

15. The government has narrowed (A)/ its list of candidates (B)/ to become the next (C)/ governor on the RBI. (D)/ No error (E)

Answer: (A)

   Directions-(Q. 16-20) Rearrange the given six sentences/group of sentences (1), (2), (3), (4), (5) and (6) in a proper sequence so as to form a meaningful paragraph and then answer the given questions :

(1) Others were shown advertisements of more affordable stuff, such as smartphones

(2) This experience shows the complexities of advertising today, when it is so easy for dissatisfied customers to make their voices heard

(3) Moreover, some of those not shown the advertisement complained, referring to themselves as, or (putting it politely losers

(4) But its advertisement was shown only to those whose profiles suggested they were potential buyers of expensive cars

(5) Eventually, this bruised a few egos

(6) Earlier this year a carmaker advertised on WeChat, a popular messaging app in China with around 550 m monthly users

16. Which of the following should be the FIRST sentence after the rearrangement?

(A)  1

(B)  3

(C)  2

(D)  6

(E)   5

Answer: (D)

17. Which of the following should be the SECOND sentence after the rearrangement?

(A)  1

(B)  2

(C)  6

(D)  4

(E)   3

Answer: (D)

18. Which of the following should be the FOURTH sentence after the rearrangement?

(A)  1

(B)  5

(C)  3

(D)  6

(E)   4

Answer: (B)

19. Which of the following should be the SIXTH (last) sentence after the rearrangement?

(A)  2

(B)  4

(C)  1

(D)  3

(E)   5

Answer: (A)

20. Which of the following should be the FIFTH sentence after the rearrangement?

(A)  5

(B)  4

(C)  1

(D)  6

(E)   3

Answer: (E)

Directions-(Q. 21-25) The sentence has two blanks, each blank indicating that something has been omitted. Choose the set of words for the blanks which best fits the meaning of the sentence as a whole.

21. The Governor’s successor will find the economy in a much better …………. Than what it was in when the himself took ……. Three years ago.

(A)  condition, holding

(B)  turmoil, over

(C)  shape, charge

(D)  characters, reigns

(E)   position, duty

Answer: (C)

22. The government’s ………. to consolidated public sector banks (PSBs) could create ……. In the current environment where stressed assets across banks are high.

(A)  actions, exposed

(B)  intend, uncertainty

(C)  proposal, secure

(D)  strategies, havoc

(E)   plans, risks

Answer: (E)

23. The IT firm may be ………… out as clients shifts to cloud services ………. By rivals.

(A)  venting, existing

(B)  losing, offered

(C)  close, provided

(D)  locking, promising

(E)   shutting, delivered

Answer: (B)

24. Inspite of ……… social performs are likely to …….. an ever larger part of marketers’ budget.

(A)  pitfalls, receive

(B)  hazards, getting

(C)  negative, share

(D)  drawbacks, obtained

(E)   fallen, have

Answer: (A)

25. Litigation and hurdles in a potential sale to a Chinese partner have ………. the company to …….. the closing of one of its plants.

(A)  raised, changing

(B)  denying, choose

(C)  forced, defer

(D)  enable, modify

(E)   compelled, defeat

Answer: (C)

Directions-(Q. 26-30) In the following passage, there are blanks, each of which has been numbered. Against each, five words are suggested, one of which fits the blank appropriately. Find out the appropriate word in each case.

   There’s been some buzz lately around the not-so-new idea that emerging technology is destroying jobs and will …(26)… destroy the middle class. Fears about a shrinking job pool are …(27)… : our economy is still recovering from the recession, and jobs have not returned to prerecession levels. Meanwhile, technology is …(28)… some low-skilled jobs. Yet, changes to the job landscape, while they may require some adjustments, are not bad news for the middle class. …(29)… advances in innovation and technology promise to make life better for everyone, both professionally and …(30)…

26. 

(A)  not

(B)  ultimately

(C)  securely

(D)  publicly

(E)   demandingly

Answer: (B)

27.

(A)  understandable

(B)  weird

(C)  unjust

(D)  remanded

(E)   wrong

Answer: (A)

28.

(A)  dicing

(B)  acquiring

(C)  replacing

(D)  lifting

(E)   paying

Answer: (C)

29.

(A)  But

(B)  Instead

(C)  Fact

(D)  Since

(E)   However

Answer: (B)

30.

(A)  workplace

(B)  shortly

(C)  personally

(D)  morosely

(E)   environment

Answer: (C)

Quantitative Aptitude

1. C is 20% more efficient than A. A and B together can finish a piece of work in 16 days. B and C together can do it in 15 days. In how many days A alone can finish the same piece of work?

(A)  42

(B)  48

(C)  54

(D)  36

(E)   45

Answer: (B)

2. A started a business with an investment of Rs 16,000. After 6 months from the start of the business, B and C joined with Rs 12,000 and Rs 18,000 respectively and A invested an additional amount of Rs 4000. If the difference between A’s share and B’s share in the annual profit is Rs 6000, what was the annual profit received?

(A)  Rs 17,600

(B)  Rs 13,200

(C)  Rs 14,300

(D)  Rs 16,500

(E)   Rs 11,000

Answer: (D)

3. Shiva gives 20% of her monthly salary to his mother, 50% of the remaining salary he invests in an insurance scheme and PPF in the respective ratio of 5 : 3 and the remaining he keeps in his bank account. If the sum of the amount he gives to his mother and that he invests in PPF is Rs 12,600, how much is Shiva’s monthly salary?

(A)  Rs 36,000

(B)  Rs 64,000

(C)  Rs 42,000

(D)  Rs 40,000

(E)   Rs 50,400

Answer: (A)

4. The respective ratio of radii of two right circular cylinders (A and B) is 4 : 7. The respective ratio of the heights of cylinders A and B is 2 : 1. What is the respective ratio of volumes of cylinders A and B?

(A)  25 : 42

(B)  23 : 42

(C)  32 : 49

(D)  30 : 49

(E)   36 : 49

Answer: (C)

5. At present, Aanshi is five years younger to Binny. Binny’s age twenty-years hence will be equal to twice of Aanshi’s age five years ago. What will be Binny’s age eight year hence?

(A)  42 years

(B)  35 years

(C)  30 years

(D)  40 years

(E)   48 years

Answer: (E)

Directions-(Q. 6-10) In this equation two equations numbered I and II are given. You have to solve both the equations and mark the appropriate option.

6. (I) 9x2 – 37x + 30 = 0

(II) 3y2 – 19y + 30 = 0

(A)  Relationship between X and Y cannot be established

(B)  X < Y

(C)  X > Y

(D)  X ≤ Y

(E)   X ≥ Y

Answer: (D)

7. (I) 2x2 – 17x + 36 = 0

(II) 5y2 – 33y + 40 = 0

(A)  Relationship between X and Y cannot be established

(B)  X < Y

(C)  X > Y

(D)  X ≤ Y

(E)   X ≥ Y

Answer: (A)

8. (I) 12x2 – 23x + 11 = 0

(II) 21y2 – 20y + 4 = 0

(A)  Relationship between X and Y cannot be established

(B)  X < Y

(C)  X > Y

(D)  X ≤ Y

(E)   X ≥ Y

Answer: (C)

9. (I) x2 + 12x + 35 = 0

(II) 7y2 + 32y + 16 = 0

(A)  Relationship between X and Y cannot be established

(B)  X < Y

(C)  X > Y

(D)  X ≤ Y

(E)   X ≥ Y

Answer: (B)

10. (I) 25x2 + 20x + 3 = 0

(II) 4y2 + 11y + 6 = 0

(A)  Relationship between X and Y cannot be established

(B)  X < Y

(C)  X > Y

(D)  X ≤ Y

(E)   X ≥ Y

Answer: (C)

11. A number is such that when it is multiplied by 6, it gives another number which is more than 168 as the original number itself is less than 168. What is 15% of the original number?

(A)  8.4

(B)  7.8

(C)  6.6

(D)  8.8

(E)   7.2

Answer: (E)

12. Dharma invested Rs P of 3 years in scheme A which offered 12% p.a. simple interest. She also invested Rs P + 400 in scheme B which offered 10% compound interest (compounded annually), for 2 years. If the amount received from scheme A was less than that received from scheme B, by Rs 304, what is the value of P?

(A)  Rs 1400

(B)  Rs 1000

(C)  Rs 1500

(D)  Rs 900

(E)   Rs 1200

Answer: (E)

Directions-(Q. 13-17) Study the following table carefully and answer the question.

13. The number of students (both male and female) who are learning freestyle in schools C and D together is what per cent less than the number of students (both male and female) who are learning the same in schools A and B together?

(A)

(B) 

(C) 

(D) 

(E) 

Answer: (D)

14. What is the average number of students learning dance forms (other than freestyle) in schools B, C and D?

(A)  206

(B)  207

(C)  204

(D)  205

(E)   201

Answer: (B)

15. Out of the number of students (both male and female) learning freestyle in school B, 5/8 are 15 years or above. If out of the total students who are 15 years or above, 30% are females, what is the number of female students learning freestyle who are below 15 years?

(A)  35

(B)  42

(C)  32

(D)  46

(E)   40

Answer: (B)

16. What is the respective ratio between the total number of male students learning freestyle in schools A and C together and the total number of female students learning the same in the same schools together?

(A)  6 : 39

(B)  6 : 13

(C)  12 : 13

(D)  6 : 11

(E)   12 : 35

Answer: (B)

17. What is the difference between the number of male students studying freestyle in schools B and D together and the total number of female students studying the same in the same schools together?

(A)  22

(B)  18

(C)  26

(D)  16

(E)   12

Answer: (A)

18. Per cent profit earned when an article is sold for 558 is double the per cent profit earned when the same article is sold for Rs 504. If the marked price of the article is 30% above the cost price, what is the marked price of the article?

(A)  Rs 585

(B)  Rs 595

(C)  Rs 624

(D)  Rs 590

(E)   Rs 546

Answer: (A)

Directions-What will come in place of question-mark (?) in the given number series?

19. 150 102 70 46 26 ?

(A)  7

(B)  13

(C)  8

(D)  2

(E)   12

Answer: (C)

20. 10 14 24 52 134 ?

(A)  351

(B)  302

(C)  368

(D)  341

(E)   378

Answer: (E)

21. 24 11 10 14 27 ?

(A)  67.5

(B)  60.5

(C)  66.5

(D)  61.5

(E)   62.25

Answer: (C)

22. 4500 900 90 6 ? 0.012

(A)  0.3

(B)  0.09

(C)  0.9

(D)  0.015

(E)   0.03

Answer: (A)

23. 8 7 12 33 128 ?

(A)  672

(B)  684

(C)  635

(D)  620

(E)   692

Answer: (C)

24. 3? × √170 = 183.998 ÷001 + 328.02

(A)  2

(B)  4

(C)  1

(D)  5

(E)   3

Answer: (E)

25. ?% of (230.02 ×89 – 559.85) = 960

(A)  20

(B)  80

(C)  50

(D)  70

(E)   75

Answer: (E)

26. 518.002 – 44.99 + 8.998 – 8.012 = ?

(A)  650

(B)  350

(C)  450

(D)  550

(E)   600

Answer: (C)

27. 358.98 ÷ 15.02 + 450.15 ÷ 8.992 + 56.02 = ?

(A)  230

(B)  200

(C)  180

(D)  150

(E)   130

Answer: (E)

28. 25% of 459 + 65.01 ÷02 = ?

(A)  109

(B)  128

(C)  234

(D)  80

(E)   186

Answer: (B)

Directions-Refer to the bar graph and answer the given question.

29. In 2005, out of total number of applications received for subjects A and B together, only 30% were accepted. What was the total number of applications accepted for subjects A and B together in 2005?

(A)  121

(B)  123

(C)  129

(D)  131

(E)   133

Answer: (B)

30. In 2004, 30% of applications received for subject A and 20% of applications received for subject B were from international students. What was the total number of International applicants for subjects A and B together in 2004?

(A)  91

(B)  97

(C)  89

(D)  93

(E)   87

Answer: (A)

31. If the respective ratio of total number of applications received for subjects A and B together in 2008 and 2009 is 3 : 4, what was the total number of applications received for subjects A and B together in 2009?

(A)  500

(B)  560

(C)  400

(D)  520

(E)   500

Answer: (B)

32. What is the average number of applications received for subject A in 2005, 2007 and 2008?

(A)  190

(B)  180

(C)  170

(D)  200

(E)   160

Answer: (A)

33. Number of applications received for subject B increase by what per cent from 2004 to 2006?

(A) 

(B) 

(C) 

(D) 

(E) 

Answer: (C)

34. In a bag, there are 8 red balls and 7 green balls. Three balls are picked at random. What is the probability that two balls are red and one ball is green in colour?

(A)  28/65

(B)  22/65

(C)  37/65

(D)  3/13

(E)   1/13

Answer: (A)

35. A vessel contains 120 litres of mixture of milk and water in the respective ratio 11 : 4. Forty-five litres of this mixture was taken out and replaced with 5 litres of water. What is the percentage of water in the resultant mixture?

(A)  35

(B)  30

(C)  25

(D)  20

(E)   15

Answer: (*)

Reasoning

   Directions-(Q. 1 to 3) Study the following information and answer the given question.

   S is the mother of D. K is the brother of D. K is the only son of M. M is the son of U. U is the husband of T. T is the mother of Y.

1. If D is married to X, then how is X related to M?

(A)  Son-in-law

(B)  Daughter-in-law

(C)  Son

(D)  Niece

(E)   Daughter

Answer: (A)

2. How is T related to K?

(A)  Mother

(B)  Aunt

(C)  Granddaughter

(D)  Grandmother

(E)   Mother-in-law

Answer: (D)

3. How is S related to Y?

(A)  Sister

(B)  Cousin

(C)  Mother-in-law

(D)  Niece

(E)   Sister-in-law

Answer: (E)

Directions-(Q. 4 to 8) In this question, relationship between different elements is shown in the statements. The statements are followed by conclusions. Study the conclusions based on the given statement and select the appropriate answer.

4. Statements :

R > S = T ≥ U; S ≥ A > V

Conclusions: (I) A < R

(II) V ≤ U

(A)  Both conclusion I and II are true

(B)  Only conclusion I is true

(C)  Neither conclusion I nor II true

(D)  Only conclusion II is true

(E)   Either conclusion I or conclusion II is true

Answer: (B)

5. Statements:

B > E ≥ A ≥ T = H < I ≤ M

Conclusions : (I) H ≤ E

(II) B > T

(A)  Both conclusion I and II are true

(B)  Only conclusion I is true

(C)  Neither conclusion I nor II is true

(D)  Only conclusion II is true

(E)   Either conclusion I or conclusion II is true

Answer: (A)

6. Statements:

S ∠M < I < T: R ≥ J > I

Conclusions : (I) R = S

(II) S < R

(A)  Both conclusion I and II are true

(B)  Only conclusion I is true

(C)  Neither conclusion I nor II is true

(D)  Only conclusion II is true

(E)   Either conclusion I or conclusion II is true

Answer: (D)

7. Statements:

B > E ≥ A ≥ T = H < I ≤ M

Conclusions: (I) E < I

(II) M ≤ A

(A)  Both conclusion I and II are true

(B)  Only conclusion I is true

(C)  Neither conclusion I nor II is true

(D)  Only conclusion II is true

(E)   Either conclusion I or conclusion II is true

Answer: (C)

8. Statements:

D ≤ O ≤ L > C ≥ E

Conclusions: (I) O < E

(II) L ≥ D

(A)  Both conclusion I and II are true

(B)  Only conclusion I is true

(C)  Neither conclusion I nor II is true

(D)  Only conclusion II is true

(E)   Either conclusion I or conclusion II is true

Answer: (D)

Directions-(Q. 9 and 10) Read the following information and answer the given question:

   Vansh starts walking from Point E and walks 25 m towards the north. He then takes a right turn and walks for 15 m. He makes a left turn and stops at Point M after walking for 20 m.

Point K is 30 m to the west of Point M. Point K is 45 m to the north of Point J. Point J is 10 m to the east of Point L.

9. How far and in which direction is Point E with respect to Point L?

(A)  30 m towards West

(B)  25 m towards East

(C)  20 m towards East

(D)  20 m towards West

(E)   25 m towards West

Answer: (B)

10. If neha is standing at Point D which is 20 m to the north to of Point L, in which direction will she have to walk in order to reach Point M?

(A)  North-west

(B)  East

(C)  South-east

(D)  North-east

(E)   North

Answer: (D)

Directions-(Q. 11 to 13) Read the given information carefully and answer the question.

   Each of the six sections, U, V, W, X, Y and Z of the same class has different number of students. Only Z has more number of students than X. V has more number of students than Y but less than U. W has more number of students than both the both Y and U. The section having the third highest number of students has 39 students. Y has 24 students.

11. If the number of students in sections W + Z is sixty-six more than the number of students in section Y, how many of students are there in section Z?

(A)  31

(B)  46

(C)  51

(D)  55

(E)   45

Answer: (C)

12. How many students does section V possibly have?

(A)  39

(B)  43

(C)  55

(D)  31

(E)   14

Answer: (D)

13. Which of the following is true regarding the number of students in section U?

(A)  No other section has less students than U

(B)  X has more number of students than U

(C)  U possibly has 45 students

(D)  U has more number of students than only three sections

(E)   None of these

Answer: (B)

Directions-(Q 14 to 19) Study the following information and answer the question.

   Seven people, namely, A, B, C, D, E, F and G have an appointment but not necessarily in the same order, on seven different months (of the same year) namely January, February, April, June, August, October and December. Each of them also likes a different activity namely Drawing, Singing, Painting, Boxing, Karate, Craft and Running but not necessarily in the same order.

The one who likes Craft has an appointment on one of the months before April. Only two people have an appointment between the one who likes craft and the one who likes painting. Only one person has an appointment between the one who likes painting and the one who likes running. The one who likes running has an appointment in a month which has 31 days. Only three people have an appointment between the one who likes running and E. G has an appointment on one of the months before E. G does not have an appointment in the month which has the least number of days. Only three people have a appointment between G and C. Only one person has an appointment between C and the one who likes Karate. The one who likes Karate has an appointment before C. The one who likes singing has an appointment immediately before B. B has an appointment in a month which has less than 31 days. Only one person has an appointment between A and F. A has an appointment before F. Only one person has an appointment between F and the who likes drawing.

14. Who amongst the following has an appointment before the one who has an appointment in December?

(A)  B

(B)  E

(C)  C

(D)  The one who likes Running

(E)   The one who likes Boxing

Answer: (E)

15. In which of the following pairs, both the persons have an appointment in months which have less than 31 days?

(A)  A, F

(B)  F, D

(C)  B, F

(D)  E, D

(E)   B, A

Answer: (E)

16. Which of the following combinations is correct?

(A)  G-Singing

(B)  A-Painting

(C)  F-Running

(D)  B-Karate

(E)   D-Painting

Answer: (A)

17. As per the given arrangement Craft is related to April and Karate is related to June following a certain pattern, which of the following is Drawing related to following the same pattern?

(A)  February

(B)  October

(C)  December

(D)  August

(E)   January

Answer: (D)

18. Who amongst the following likes Running?

(A)  F

(B)  D

(C)  B

(D)  A

(E)   Other than those given as options

Answer: (B)

19. How many people have an appointment between the one who has an appointment in February and A?

(A)  Three

(B)  None

(C)  More than three

(D)  One

(E)   Two

Answer: (D)

Directions-(Q. 20 to 24) Study the given information carefully to answer the given question.

   Seven boxes-A, B, C, D, E, F and G are kept one above the other, but not necessarily in the same order. Each box contains different items-Shoes, Papers, Bands, Medicines, Ribbons, Creams and Phones, but  not necessarily in the same order.

Only three boxes are kept between D and G. The Ribbon box is kept immediately above G. Only one box is kept between in Ribbon box and A. The Ribbon box is not the second from the bottom of the stack. Only one box is kept between E and A. E is kept above A. The Medicine box is kept immediately above E. Only three boxes are kept between the Medicine box and the Shoe box. The Paper box is immediately above the Phone box. G is not the Paper box. F is kept immediately below the Cream box. Only one box is kept between B and the Cream box.

20. Four of the following five are alike in a certain way and hence from a group. Which of the following does not belong to the group?

(A)  G-Creams

(B)  E-Bands

(C)  C-Shoes

(D)  D-Papers

(E)   A-Bands

Answer: (C)

21. Which of the following boxes contains bands?

(A)  D

(B)  C

(C)  G

(D)  A

(E)   Other than those given as options

Answer: (C)

22. What is the position of F in the given stack of boxes?

(A)  Second from the top

(B)  Third from the bottom

(C)  First from the top

(D)  Fifth from the bottom

(E)   Fourth from the top

Answer: (B)

23. Which of the following boxes is kept immediately below G?

(A)  A

(B)  B

(C)  C

(D)  The Shoe box

(E)   The Paper Box

Answer: (A)

24. How many boxes are kept between B and the Ribbon box?

(A)  Two

(B)  One

(C)  None

(D)  Three

(E)   More than three

Answer: (D)

Directions-(Q. 25 to 29) Study the following information to answer the given question.

   Eight friends, C, D, E, F, L, M, N and O are seated in a straight line, but not necessarily in the same order. Some of them are facing north while some face south.

Only three people sit to the right of M. E sits second to the left of M.

F sits third to the right of O. O is not an immediate neighbor of M. O does not sit at any of the extreme ends of the line.

Both the immediate neighbours of O face south.

D sits second to the right of N.

As many people sit between M and D as between M and L.

Immediate neighbours of F face opposite direction (i.e., if one person faces north then the other person faces south and vice-versa.)

L and F face direction opposite to C (i.e. If C faces north then both L and F face south and vice-versa.)

25. Which of the following is true, based on the given arrangement?

(A)  D faces north

(B)  Only three people face south

(C)  L sits at one of the extreme ends of the line

(D)  O and E face the same directions

(E)   None of the given options is true

Answer: (D)

26. How many people sit to the left of O?

(A)  Three

(B)  More than four

(C)  One

(D)  Four

(E)   Two

Answer: (C)

27. Who amongst the following faces north?

(A)  E

(B)  M

(C)  F

(D)  N

(E)   D

Answer: (A)

28. Who amongst the following sits second to the left of L?

(A)  O

(B)  F

(C)  D

(D)  No one as less than two people sit to the left of L

(E)   N

Answer: (D)

29. Who among the following represent the persons sitting at extreme ends of the line?

(A)  D, N

(B)  C, D

(C)  L, N

(D)  D, L

(E)   C, N

Answer: (B)

Direction-(Q. 30-35) Study the following information and answer the given question.

   Seven people, namely A, B, C, D, E, F and G teach seven different subjects namely, Mathematics, English, Chemistry, History, Accountancy, Physics and Biology. Each of them works in either of the three institutes viz., Paramount, Brilliant and Embibe atleast two of them in a institute.

(Note: None of the information given is necessarily in the same order.)

   G teaches in Brilliant with the one who teaches Accountancy. The one who teaches Biology works in an institute with only the one who teaches Chemistry. B teaches English. B does not work with G. D works with F but not in Embibe. Neither D nor F teaches Accountancy. F does not teach Chemistry. C works with only the one who teaches Mathematics. E works with the one who teaches History.

30. Who amongst the following teaches Physics?

(A)  F

(B)  D

(C)  C

(D)  E

(E)   G

Answer: (D)

31. Which of the following combinations represents the institute in which C works and the subject he teaches?

(A)  Brilliant-Accountancy

(B)  Embibe-Accountancy

(C)  Paramount-Biology

(D)  Brilliant-History

(E)   Paramount-Chemistry

Answer: (A)

32. Which of the following subjects does a teach?

(A)  Biology

(B)  Chemistry

(C)  Mathematics

(D)  Accountancy

(E)   History

Answer: (E)

33. Which of the following statements is true?

(A)  D teaches Biology

(B)  None of the given statements is true

(C)  Only two people teach in brilliant

(D)  Both E and G work in the same institute

(E)   A teaches Mathematics

Answer: (C)

34. Which of the following combination represents the combination of people working in Paramount?

(A)  D and the one who teaches English

(B)  E, A

(C)  G, C, E

(D)  F and the one who teaches Chemistry

(E)   A, B

Answer: (D)

35. Four of the following five are alike in a certain way based on the given arrangement and hence form a group. Which of the following does not belong to that group?

(A)  BE

(B)  GC

(C)  EA

(D)  FG

(E)   AB

Answer: (D)

State Bank of India PO Preliminary Exam-2017 Question Paper With Answer Key

State Bank of India PO Preliminary Exam-2017
State Bank of India PO Preliminary Exam-2017 Question Paper With Answer Key

State Bank of India PO Preliminary Exam-2017

Solved Paper

English Language

Directions-(Q. 1-10) Read the following passage carefully and answer the questions given below it. Certain words have been printed in bold to help you locate them while answering some of the questions.

   On January 19th every year, Americans celebrate the life and achievements of Dr. Martin Luther King Jr. (MLK), a Baptist clergyman turned activist, who is often credited with spearheading the country’s civil rights movement. Thanks to his efforts that America is today a nation where everyone has equal rights, regardless of race, colour or creed. MLK who would have celebrated his 86th birthday on January 15th, 2015, was born in Atlanta, Georgia, in 1929, a city where racial divide was a way of life. Blacks and whites lived totally disparate lives. They went to different schools, ate at different restaurants and even sat on specially designated seats on buses and trains. Though this did not seem right to the young boy, just like the others, he accepted it as a way of life. Things started to change in the summer of 1944 when 15 year old MLK left Atlanta to work in the tobacco fields of Simsbury, Connecticut. To his surprise, the black residents in the Northern states were not subjected to the same kind of racial injustice. The young boy expressed his astonishment in a letter he wrote to his father in June saying, “After we passed Washington there was no discrimination at all. The white people here are very nice. We can go to any place we want to and sit any where we want to.” By the end of the summer, the seeds of what would transform MLK into America’s most influential civil rights leader had been firmly planted. In 1954, MLK who was by now an ordained Minister and married chose to become pastor of a church in Montgomery, Alabama, a city that was notorious for its racial discrimination. His foray into activism began gradually-by encouraging Montgomery residents to register to vote and join the NAACP, the nation’s oldest and largest civil rights organization. However, his passive stance changed on December 1st, 1955 after fellow activist Rosa Parks was arrested for refusing to vacate her seat on a public bus for a white passenger. Enraged, MLK asked the black residents of Montgomery to boycott all public transportation. This was no easy request. The buses were the only commute mode for the residents, whose jobs often entailed traveling long distances. But they took the challenge not for just one day or month, but a full year ! As the news of the boycott spread, black people from other parts of the nation that had similar laws, joined in ! In 1956, the activist won their first battle when the Supreme Court of the United States passed a ruling that abolished the transportation segregation law. But MLK was just getting started. He decided to dedicate his life to the cause and spent the next decade travelling around the country, spurring all Americans to stand up against segregation in a non-violent peaceful manner by organizing sitins, boycotts and protest marches. While he gave many inspiring speeches, his most memorable one was delivered on August 28th, 1963. The events leading to the oft-quoted ‘I have a dream’ speech began in June of that year when President John F. Kennedy asked the US Congress to pass a civil rights bill-one that would give all Americans equal access to public places. To convince government officials to pass the bill, MLK along with other civil rights leaders asked people to demonstrate their support by staging a peaceful march in Washington D.C. Over 250,000 Americans from all over the country flew, drove, rode buses and even walked, to participate in what the history books now call the March on Washington ! It was at this event while standing on the steps of the Lincoln Memorial, that MLK articulated his dream of living in a country where everyone was treated equally. Unfortunately, the civil rights activist was killed while on a trip to Memphis, Tennessee in 1968, and did not live long enough to see it come true. But had MLK been alive, he sure would have been proud to see how far the country has come in recognizing that every-one is equal regardless of race, color creed !

1. Why did he initially accept discrimination as a way of life?

(A)  As he was afraid of the whites

(B)  As his father wanted him to accept that as a form of life

(C)  So as to save himself from the abuses and the beatings by the whites

(D)  Because he lacked decision making capabilities as he was young

(E)   As he was ignorant to the cause of the blacks

Answer: (D)

2. What happened when he went to Simsbury in the summer of 1944?

(A)  It was the first time he saw specially designated seats on buses and trains for blacks

(B)  His passive stance changed

(C)  He was stupefied by what he saw there

(D)  He encouraged the residents there to vote

(E)   He gave his famous speech ‘I hav a dream’

Answer: (C)

3. Why was MLK enraged?

(A)  Because of specially designated bus seats for black people in Montgomery

(B)  As a fellow activist was apprehended

(C)  Racial injustice in Washington

(D)  Different schools on the basis of race

(E)   He was enraged with how people accepted the discrimination

Answer: (B)

4. What was the first victory won by the activists?

(A)  Passing of the civil right bill

(B)  The voting rights granted to the black people

(C)  Abolishment of transportation segregation law

(D)  Boycotting public transportation by the black people

(E)   Right to vote

Answer: (C)

5. What led to the famous “I have a dream” speech?

(A)  The arrest of the activist named Rosa Parks

(B)  Kennedy’s urge to fellow congress to pass the civil right bill

(C)  Transportation segregation law

(D)  US congress’ reluctance to pass the bill

(E)   None of the above

Answer: (B)

6. What was MLK’s dream?

(A)  Abolition of public transportation segregation law

(B)  Right to vote

(C)  Equal access in all public places

(D)  To exonerate Rosa parks from the accusation of violating civil rights

(E)   He wanted a person who is black to be the next U.S. President

Answer: (C)

   Directions-(Q. 7 and 8) Which of the following words is nearest in the meaning to words as given in bold letters?

7. Entail

(A)  Engender

(B)  Prohibit

(C)  Exclude

(D)  Feign

(E)   Dissuade

Answer: (A)

8. Foray

(A)  Exodus

(B)  Retreat

(C)  Incursion

(D)  Disengagement

(E)   Detachment

Answer: (C)

   Directions-(Q. 9 and 10) Which one of the following words is most opposite in meaning to the words given in bold letter in the passage?

9. Ordained

(A)  Appoint

(B)  Consecrate

(C)  Sanctify

(D)  Abrogate

(E)   Enact

Answer: (D)

10. Spurring

(A)  Incentive

(B)  Stimulant

(C)  Inducement

(D)  Impetus

(E)   Deterrent

Answer: (E)

   Directions-(Q. 11-15) Rearrange the following seven sentences (a), (b), (c), (d), (e), (f) and (g) in the proper sequence to form a meaningful paragraph and then answer the questions given below.

(a) One is the adverse global environment.

(b) But the second can be tackled and should have been tackled by now. Unfortunately, precious time has been lost for want to clarity on the way forward.

(c) It will be up to the incoming RBI governor, therefore, to show the way. The bulk of the bad loans is with public sector banks (PSBs).

(d) As bad loans mounted and banks had to make provisions for these, profits fell at PSBs or losses mounted. PSB performance suffered in comparison with private banks.

(e) The other is a banking sector weighed down by bad loans. There isn’t much we can do about the first.

(f) The Reserve Bank of India (RBI) is better placed than anybody else to feel the pulse of the banking sector.

(g) The Modi government’s attempts to accelerate growth and realize the promise of achche din face two formidable obstacles today.

11. Which of the following should be the SECOND sentence after the rearrangement?

(A)  a

(B)  b

(C)  f

(D)  d

(E)   e

Answer: (A)

12. Which of the following should be the FIRST sentence after the rearrangement?

(A)  a

(B)  c

(C)  b

(D)  g

(E)   e

Answer: (D)

13. Which of the following should be the FIFTH sentence after the rearrangement?

(A)  e

(B)  d

(C)  b

(D)  f

(E)   a

Answer: (D)

14. Which of the following should be the FOURTH sentence after the rearrangement?

(A)  a

(B)  b

(C)  c

(D)  f

(E)   e

Answer: (B)

15. Which of the following should be the SIXTH sentence after the rearrangement?

(A)  c

(B)  d

(C)  a

(D)  b

(E)   e

Answer: (A)

   Directions-(Q. 16-20) The following questions have two blanks, each blank indicating that something has been omitted. Choose the set of words for each blank that best fits the meaning of the sentence as a whole.

16. Though the country has ……. Free medical service for the poor. It is ……

(A)  stopped, unaffordable

(B)  rendered, essential

(C)  maintained, admirable

(D)  favoured, appreciable

(E)   instituted, inadequate.

Answer: (E)

17. If India is …….. on protecting its resources, international business appears equally …………… to safeguard its profit.

(A)  dreaded, fragile

(B)  Stubborn, weak

(C)  Bent, determined

(D)  Approaching, settled

(E)   Obsessed, prepared

Answer: (C)

18. Obesity and uncontrolled diet…….. together to…….. the risk of liver disease in both men and women.

(A)  Act, increase

(B)  Result, aggravate

(C)  Taken, arrest

(D)  Put, heighten

(E)   Mix, lower

Answer: (A)

19. The blame game for the air tragedy is already in full …… with the authorities involved making attempts to ….. themselves up.

(A)  Sway, defend

(B)  View, try

(C)  Fledged, protect

(D)  Swing, cover

(E)   Roll, hide

Answer: (D)

20. Human beings are biological …….. they need to struggle in order to …….

(A)  Things, forward

(B)  Souls, back

(C)  Organisms, survive

(D)  Features, undone

(E)   Creatures, again

Answer: (C)

   Directions-(Q. 21-25) Read each sentence to find out whether there is any grammatical error in it. The error, if any, will be one part of the sentence. The number of that part is the answer. If there is no error, the answer is (E). (Ignore the errors of punctuation, if any).

21. In Indian working woman (A)/ lead a life of dual responsibilities (B)/ if they are married (C)/ and have a family. (D)/ No error (E)

Answer: (A)

22. Though, she has aptitude (A)/ in mathematics, I won’t allow (B)/ her to take it up as a subject of study for the Master’s Degree (C)/ because I know the labour involved will tell upon her health. (D)/ No error (E)

Answer: (E)

23. The RBI has proposed to introduce (A)/ polymer notes after taking into considering (B)/ the cost and longevity (C)/ associated with their manufacturing. (D)/ No error (E)

Answer: (B)

24. Over the next five years (A)/ the government needs to invest (B)/ at less 350 billion dollars (C)/ in rural infrastructure. (D)/ No error (E)

Answer: (C)

25. Money from poor countries (A)/ is flowing into richer ones (B)/ in large part due to the active purchase (C)/ of foreign assets by central banks. (D)/ No error (E)

Answer: (A)

   Directions-(Q. 26-30) In the following passage, some of the words have been left out, each of which is indicted by a number. Find the suitable word from the options given against each number and fill up the blanks with appropriate words to make the paragraph meaningful.

   Rural healthcare in India is ….(26)… by a huge gap between supply and demand. Currently, rural healthcare needs met either by limited government facilities and private nursing homes, which have not been able to keep pace with increasing demand or by a number of quacks who practice medicine in rural areas. The quality of infrastructure is usually poor and people end up having to go to nearby large cities if they need high quality care. Rural India deserves, better, since the ability to pay has gone up over the last few years, driven by growth in income penetration of government healthcare programmes. Increasing demand …(27)… with the failure of existing infrastructure to scale, has resulted in rural healthcare …(28)… a large under served market. Absence of a viable business model …(29)… conversion of the huge rural expenditure on health into an economic activity that generates income and serves the poor. It is this …(30)… that entrepreneurs are looking to plug.

26.

(A)  Performed

(B)  Displayed

(C)  Furthered

(D)  Characterized

(E)   Male

Answer: (D)

27.

(A)  Couple

(B)  Combined

(C)  Mentioned

(D)  Engaged

(E)   Resulting

Answer: (B)

28.

(A)  Become

(B)  Happening

(C)  Being

(D)  Exists

(E)   Is

Answer: (C)

29.

(A)  To

(B)  Makes

(C)  So

(D)  Ceasing

(E)   Prevents

Answer: (E)

30.

(A)  Gap

(B)  Truth

(C)  Progress

(D)  Catastrophes

(E)   Divides

Answer: (A)

Quantitative Aptitude

   Directions-(Q. 31-35) What will come in place of the question mark (?) in each of the following series?

31. 14, 8, 9, 14.5, 30, ?_

(A)  75

(B)  76

(C)  60

(D)  65

(E)   80

Answer: (B)

32. 20, 29, 54, 103, 184, ?

(A)  310

(B)  350

(C)  305

(D)  315

(E)   320

Answer: (C)

33. 7, 8, 18, 57, ?, 1165

(A)  250

(B)  234

(C)  230

(D)  232

(E)   235

Answer: (D)

34. 5, 7, 18, 47, 103, ?

(A)  195

(B)  210

(C)  200

(D)  190

(E)   220

Answer: (A)

35. 77, 85, 69, 101, 37, ?

(A)  180

(B)  165

(C)  170

(D)  120

(E)   175

Answer: (B)

   Directions-(Q. 36-40) Read the following line graph and answer the following questions.

36. If the sale of company X in year 2016 is increased by 20% with respect to year 2015 and the sale of company Y in years 2016 with respect to year 2015 is decreased by 10%, then find the total sale of the company X and Y together in year 2016?

(A)  7200

(B)  9600

(C)  8400

(D)  5600

(E)   8800

Answer: (B)

37. Find the ratio of the sales of company X in years 2011, 2013 and 2015 together to the total sale of company Y in year 2012 and 2014 together?

(A)  23 : 14

(B)  14 : 23

(C)  11 : 29

(D)  29 : 11

(E)   23 : 11

Answer: (A)

38. Total cars sold by both companies in year 2012 are what per cent more/less than the total cars sold by both companies in year 2013?

(A)  28%

(B)  18%

(C)  25%

(D)  20%

(E)   21%

Answer: (D)

39. Find the difference between the average number of cars sold by company X from 2011 to 2015 and the average number of cars sold by company Y from 2011 to 2015?

(A)  750

(B)  900

(C)  800

(D)  850

(E)   830

Answer: (C)

40. Find the total number of cars sold by both companies from year 2012 to 2014?

(A)  23000

(B)  21000

(C)  22500

(D)  21500

(E)   22000

Answer: (E)

   Directions-(Q. 41-45) Read the following table and answer the following questions.

41. Total number of female visitors from district Q and R together to see the museum are how much more/less than total number of male visitors from district R and S together to see the museum?

(A)  142

(B)  126

(C)  128

(D)  131

(E)   136

Answer: (D)

42. Average number of visitors from district P, Q and R together to see the museum are approximately what per cent of the average number of visitors from district S, T and Y together to see the museum?

(A)  71%

(B)  76%

(C)  78%

(D)  74%

(E)   85%

Answer: (B)

43. Find the ratio of the male visitors from district T and U together to see the museum to the female visitors form district R and S together to see the museum?

(A)  107 : 117

(B)  106 : 111

(C)  111 : 116

(D)  117 : 107

(E)   117 : 106

Answer: (C)

44. Male visitors from district R to see the museum are what percent more/less than the female visitors from district T to see the museum (Approximate up to two decimal points)?

(A) 

(B)   

(C)   

(D)   

(E)   50%

Answer: (D)

45. Find the difference between the total number of male visitors from district Q, R and S together to see the museum and the total number of female visitors from district S, T and U together to see the museum?

(A)  25

(B)  75

(C)  60

(D)  50

(E)   59

Answer: (E)

   Directions-(Q. 46-50) In the following question two equations numbered I and II are given you have to solve both the equations and given answers if.

(A) x > y                        (B) x ≥ y

(C) x < y             (D) x ≤ y

(E) x = y or the relation can not be established

46. (I) x2 – 3x + 2 = 0

(II) 2y2 – 7y + 6 = 0

Answer: (D)

47. (I) 3x2 + 4x + 1 = 0

(II) y2 + 5y + 6 = 0

Answer: (A)

48. (I) 2x2 + 5x + 2 = 0

(II) y2 + 9y + 20 =  0

Answer: (A)

49. (I) x2 – 7x + 10 = 0

(II) y2 – 12y + 35 = 0

Answer: (D)

50. (I) (x2 – 12)2 = 0

(II) y2 = 144

Answer: (E)

   Directions-(Q. 51-55) What approximate value should come in place of the question mark (?) in the following questions?

   Note-You are not expected to calculate the exact value.

51. 23.001 × 18.999 × 7.998 = ?

(A)  4200

(B)  3000

(C)  3500

(D)  4000

(E)   2500

Answer: (C)

52.

(A)  81

(B)  72

(C)  169

(D)  121

(E)   144

Answer: (C)

53. 94.95 × 13.03 + √35.98 × 14.99 = 53 × √?

(A)  25

(B)  144

(C)  225

(D)  625

(E)   900

Answer: (D)

54. 1884.88 ÷ 144.921 + 6.99 + (?)2 = 69.09

(A)  3

(B)  4

(C)  7

(D)  6

(E)   5

Answer: (C)

55. 41% of 601 – 250.17 = ? – 77% of 910

(A)  800

(B)  500

(C)  690

(D)  760

(E)   550

Answer: (C)

56. An article is marked up 40% higher than CP but it was sold on x% discount. The shopkeeper thus gains 12%. What would be the S.P. of the article with C.P. Rs 120 and sold on x% profit?

(A)  Rs 134.50

(B)  Rs 144

(C)  Rs 128

(D)  Rs 148

(E)   Rs 157

Answer: (B)

57. There are 27 cards having number 1 to 27. Two cards are picked at random one by one. What is the probability that sum of number of these 2 cards is odd?

(A)  13/27

(B)  8/13

(C)  182/729

(D)  14/27

(E)   15/32

Answer: (D)

58. B is 20% more efficient than A. B started the work and do it for x days. And then B is replaced by A. A completed the remaining work in x + 8 days. Ratio of work done by A and B is 3 : 2. In how many days A and B working together complete the whole work?

(A)    

(B)   

(C)    

(D)   

(E)     

Answer: (C)

59. A sum of Rs 9100 is borrowed at 20% per annum compounded annually. If the amount is to be paid in two years, what will be the total amount?

(A)  Rs 13,104

(B)  Rs 13,280

(C)  Rs 13,250

(D)  Rs 14,230

(E)   Rs 13,514

Answer: (A)

60. A man spends 28% of his salary on food. From the remaining he spend 1/6th on rent and sends 3/8th to his mother. If he left with Rs 5280, what amount he sends to his mother?

(A)  Rs 4230

(B)  Rs 4320

(C)  Rs 4580

(D)  Rs 4420

(E)   Rs 4570

Answer: (B)

61. The average age of a husband and wife was 23 years when they were married 5 years ago. The average age of husband, the wife and a child who was born during the interval, is 20 years now. How old is the child now?

(A)  9 months

(B)  1 year

(C)  3 years

(D)  4 years

(E)   6 years

Answer: (D)

62. The ratio between the ages of a father and a son at present is 5 : 2 respectively. Four year hence the ratio between the ages of the son and his mother will be 1 : 2 respectively. What is the ratio between the present ages of the father and the mother respectively?

(A)  3 : 4

(B)  5 : 4

(C)  4 : 3

(D)  Cannot be determined

(E)   6 : 5

Answer: (D)

63. Total distance between A and B is d If the distance travelled along the stream is three times of the total distance and the distance travelled against the stream is two times of the total distance. The time taken to cover distance along the stream is 10% less than the time taken to cover the distance against the stream. If a person covers a distance of 21 km in 1 hr. 24 min. along the stream, then find the rate of current-

(A)  2 km/hr

(B)  3 km/hr

(C)  1 km/hr

(D)  4 km/hr

(E)   5 km/hr

Answer: (B)

64. P and Q started a business by investing Rs 15,000 and Rs 18,000 respectively. After four months R joined the business with a capital of Rs 10,000. After two more months Q left the business with his capital. At the end of the year P got a share of Rs 4,500 in the profit. What is the total profit earned?

(A)  Rs 6800

(B)  Rs 7600

(C)  Rs 8600

(D)  Rs 9200

(E)   Rs 9600

Answer: (D)

65. Inside a square plot a circular garden is developed which exactly fits in the square plot and the diameter of the garden is equal to the side of the square plot which is 8 metre. What is the area of the space left out in the square plot after developing the garden?

(A)  98 m2

(B)  146 m2

(C)  84 m2

(D)  168 m2

(E)   115 m2

Answer: (D)

Reasoning Ability

   Directions-(Q. 66-70) Study the following information carefully and answer the given questions.

   In an apartment, 8 persons i.e., D, E, F, G H, I J and K live on different floors of 8 storey-building but not necessarily in the same order. The lowermost floor of the building is numbered 1 and the topmost floor of the building is numbered 8. They are of different stream of engineering i.e., Chemical Engineering, Instrumentation Engineering, Software Engineering, Aeronautical Engineering, Mechanical Engineering, Electrical Engineering, Automobile Engineering and Civil Engineering.

   The one who lives on fourth floor is specialized in Mechanical Engineering. D lives on odd numbered floor but above 3rd floor. The number of person between D and the one who is specialized in Electrical Engineering is same as number of persons between D and I. The one who is specialized in Instrumentation Engineering lives on lower most floor. K lives on an even numbered floor and he is specialized in Automobile engineering. There are two floors between E and H and E lives above to H. E is specialized in Aeronautical Engineering. J lives just above the one who is specialized in Aeronautical Engineering. The number of floors between the one who is specialized in Aeronautical Engineering and Mechanical Engineering is two. The one who is specialized in Civil engineering lives on odd numbered floor. The number of floors between the one who is specialized in Chemical Engineering and J is four. The one who is specialized in Aeronautical Engineering lives on an odd numbered floor. The number of floors between the one who is specialized in Civil Engineering the floor on which F lives in same as the number of floors between F and G. I lives below the floor on which D lives.

66. How many persons live between the person who is specialized in Chemical Engineering and the one who is specialized in Electrical engineering?

(A)  Six

(B)  One

(C)  Four

(D)  Two

(E)   None of these

Answer: (C)

67. J is specialized in which of the following stream of Engineering?

(A)  Aeronautical Engineering

(B)  Electrical Engineering

(C)  Civil Engineering

(D)  Instrumentation Engineering

(E)   Mechanical Engineering

Answer: (B)

68. Four of the following five are alike in a certain way and hence they from a group. Which one of the following does not belong to that group?

(A)  J

(B)  K

(C)  H

(D)  I

(E)   G

Answer: (E)

69. D lives on which floor?

(A)  1

(B)  3

(C)  4

(D)  5

(E)   7

Answer: (D)

70. G is related to Mechanical Engineering, in the same way as F is related to Automobile Engineering. Then, which of the following is H related to? (Following the same pattern)

(A)  Mechanical Engineering

(B)  Electrical Engineering

(C)  Civil Engineering

(D)  Instrumentation Engineering

(E)   Aeronautical Engineering

Answer: (E)

71. Which of the following symbols should replace the sign ($) and (#) in the given expression in order to make the expressions P > C and C ≤ B definitely true?

‘A > B ≥ R $ C < R ≤ Z = M # P ≥ X’

(A)  ≥, >

(B)  ≥, ≤

(C)  >, =

(D)  =, ≥

(E)   <, ≤

Answer: (B)

72. Five persons namely A, B, C, D and E are going to the school in different days of the week, starting from Monday to Friday. Two persons are going between C and B. C is going before Wednesday. D is going to the school immediate after E. A is not going on Friday. Then who among the following persons are going to school on Wednesday?

(A)  B

(B)  C

(C)  D

(D)  E

(E)   A

Answer: (D)

   Directions-(Q. 73-77) Study the following information carefully and answer the given questions.

   Eight people viz., A, B, C, D, P, Q, R and S are sitting in a straight line. They all are facing north. Each one of them has a different age i.e., 14, 16, 17, 19, 21, 23, 26 and 31 year, but not necessarily in the same order.

   B sits at one of the extreme end of the row. There are three persons sitting between C and Q. Q is of neither 14 nor 19 years old. There are three persons sitting between D and the person whose age is 23 years. Neither Q nor D is the oldest person. Age difference of immediate neighbours of D is 5 years. A sits to the right of R, but not immediate right. There are three persons sitting between B and the one whose age is 16 years. The one whose age is 19 years sits third to the right to C. R sits to the right of B. Q sits second to the right of the person whose age is 23 years. P sits immediate left of the person whose age is 14 years. Q is not youngest person. The one, whose age is 31 years in not immediate neighbor of the youngest person, C is not the fourth oldest person.

73. Who sits second to the right of D?

(A)  A

(B)  S

(C)  P

(D)  R

(E)   None of these

Answer: (E)

74. How many persons sits between the persons whose age is 31 years and S?

(A)  Four

(B)  Five

(C)  Three

(D)  One

(E)   None of these

Answer: (A)

75. Who among the following person is 26 years old?

(A)  R

(B)  D

(C)  C

(D)  S

(E)   None of these

Answer: (E)

76. If P is related to 16 years in the same way as B is related to 26 years, then which of the following is R related to, following the same pattern?

(A)  19 years

(B)  17 years

(C)  21 years

(D)  31 years

(E)   None of these

Answer: (A)

77. What is the age difference of A’s immediate neighbours?

(A)  Three

(B)  Seven

(C)  Five

(D)  Six

(E)   None of these

Answer: (D)

78. Q is the daughter of A. J is the brother of Q. J is the son of R. J is the father of S. If it is given that A is mother of Q, then what is the relation of R with respect to S?

(A)  Father

(B)  Mother-in-law

(C)  Mother

(D)  Father-in-law

(E)   Grandfather

Answer: (E)

   Directions-(Q. 79-83) Study the following information carefully and answer the given questions.

   There are seven persons P, Q, R, S, T, U and V who were born on the same day of the same month of different year i.e., 1984, 1946, 1967, 1972, 1982, 1989 and 1992 but not necessarily in the same order.

   Note : Their age are considered as on the same month and day of 2017 as their date of births.

   The difference between the ages of Q and R is twice the square root of the age of one of the any seven persons. Difference between the ages of R and S is the same as the number obtained by dividing ages of any of the other five persons. Age of P is greatest amongst those whose age is a multiple of five. T is older than V who is not the youngest. Q is not youngest person.

79. Who amongst the following person is the oldest?

(A)  P

(B)  V

(C)  U

(D)  T

(E)   None of these

Answer: (D)

80. What is the age of R?

(A)  33  years

(B)  35 years

(C)  25 years

(D)  45 years

(E)   50 years

Answer: (B)

81. How many persons are youngest than U?

(A)  One

(B)  Two

(C)  Three

(D)  Four

(E)   None of these

Answer: (E)

82. What is the age of S?

(A)  45 years

(B)  35 years

(C)  33 years

(D)  50 years

(E)   28 years

Answer: (C)

83. Who was born on 1989?

(A)  V

(B)  U

(C)  T

(D)  P

(E)   Q

Answer: (A)

84. In a certain code language ‘economics growth registered’ is written as ‘ve jo qi’, ‘growth is expected’ is written as ‘qi lo mn’, and ‘registered expected number’ is written as ‘lo ve pr’, then what is the code for “economic”?

(A)  lo

(B)  pr

(C)  qi

(D)  ve

(E)   jo

Answer: (E)

85. If 2 is subtracted from each odd digit in the number 7493652 and 3 is added to each even digit in number then which of the following digit is repeated in the new number so obtained?

(A)  9, 3

(B)  6, 5

(C)  5, 4

(D)  5, 9

(E)   5, 7

Answer: (E)

   Directions-(Q. 86 and 87) Study the following information carefully and answer the given below :

   There are four boxes i.e.., J, K, L and M in which four types of fruits are stored. Fruits re Litchi, Apple, Grapes and Mango, Boxes are arranged in such a manner from top to bottom.

   There are two boxes between K and L. The box is which grapes are stored is above L, but not immediate above. The box in which Apple is stored is immediate below M, but not stored in box L. Litchi box is above is Mango box, but not immediate above Apple box.

86. In which of the following box, Litchi is stored?

(A)  J

(B)  M

(C)  K

(D)  L

(E)   Either J or M

Answer: (C)

87. Which of the following fruit is stored in second lowest Box?

(A)  Grapes

(B)  Apple

(C)  Mango

(D)  Litchi

(E)   Cannot be determined

Answer: (B)

   Directions-(Q. 88-92) Study the following information carefully and answer the given below :

   Eight persons M, N, O, V, W, X, Y and Z attend seminars on different months of the y ear viz., March, June, October and November, such that not more than two persons attend their seminars in each of the months. Seminars can be held on either 10thh or 27th day of the month. No two seminars can be held on the same day.

   W and N attend the seminars on the same month. There are three seminars between the seminars of X and O. W does not attend their seminar in November. Z attends his seminar immediately after N. V attends his seminar in the month of November. The number of persons who attend their seminars between the seminars Y and Z respectively is the same as the number of persons who attend their seminars between the seminars of N and V respectively. X does not attend the seminar on October. W attends his seminar before N.

88. M attends his seminar on which of the following dates?

(A)  10th October

(B)  27th November

(C)  10th November

(D)  10th March

(E)   None of these

Answer: (C)

89. Which of the following persons attends his seminar on 27th March?

(A)  W

(B)  N

(C)  M

(D)  X

(E)   None of these

Answer: (D)

90. How many persons attend the seminar after W?

(A)  5

(B)  4

(C)  6

(D)  3

(E)   None of these

Answer: (A)

91. Who among the following persons attends the seminar on 10th October?

(A)  W

(B)  M

(C)  V

(D)  Z

(E)   None of these

Answer: (D)

92. How many persons attended seminar after V?

(A)  5

(B)  4

(C)  7

(D)  No one

(E)   None of these

Answer: (D)

   Directions-(Q. 93 and 94) Study the following information carefully and answer the given questions:

   There are six family members A, B, C, D, E and F and all of them are different ages. A is younger than only one person. E is older than B and D but not as old as A. D is older than only one person. F is youngest in the family. The age of D is 25 years and the age of person who is second oldest is 40  years.

93. Who is the oldest in the family?

(A)  A

(B)  B

(C)  D

(D)  E

(E)   C

Answer: (E)

94. What is the possible age of B?

(A)  42 years

(B)  20 years

(C)  55 years

(D)  19 years

(E)   30 years

Answer: (E)

95. Which of the following statements shows ‘A ≥ R’ and ‘B < C’ holds definitely true?

(A)  B ≤ C = A ≥ K = R

(B)  C = K > B < R ≥ A

(C)  C > B > A ≥ K = R

(D)  B = K < C < R = A

(E)   None of these

Answer: (C)

   Directions-(Q. 96-98) In this question, relationship between different elements is shown in the statements. The statement is followed by conclusions. Study the conclusions based on the given statement and select the appropriate answer.

(A) Only conclusion I is implicit.

(B) Both conclusion I and II are implicit

(C) Either conclusion I or II is implicit

(D) Only conclusion II is implicit

(E) Neither conclusion I nor II is implicit

96. Statements : M > U > L ≤ N; L ≥ Y > A

Conclusion I : Y < N

Conclusion II : Y = N

Answer: (C)

97. Statements : J ≥ A > D = E; L < A < M

Conclusion I : M < J

Conclusion II : J > L

Answer: (D)

98. Statements : M ≤ K > L = Y ; P ≤ T > M

Conclusion I : P > Y

Conclusion II:  T < L

Answer: (E)

99. In a vertical row 13 persons are sitting. A is seventh from the beginning and two persons sits between G and A. Persons between A and L is same as persons between G and Q. Then what is the position of Q from the beginning?

(A)  Fourth

(B)  Eighth

(C)  Sixth

(D)  Ninth

(E)   Cannot e determined

Answer: (E)

100. A man walks 12 m east from point A and reaches point B. From point B he takes turn and walked 6 m and again he takes right turn and walks 7 m and again takes right turn and reaches point M. If it is given that point B is in north from point M, then what is the distance between B and M?

(A)  7 m

(B)  6 m

(C)  5 m

(D)  4 m

(E)   3 m

Answer: (E)

© Copyright Entrance India - Engineering and Medical Entrance Exams in India | Website Maintained by Firewall Firm - IT Monteur